Question From (1 To 50 )
1. A 32-week pregnant woman with a risk of premature birth is being treated for fetal respiratory distress syndrome. What drug is prescribed? Select one:
A. Misoprostol
B. Oxytocin
C. Ginipral
D. Progesterone
E. Dexamethasone
The correct answer is: Dexamethasone
Explanation
The drug that is typically prescribed to prevent fetal respiratory distress syndrome in a pregnant woman at risk of premature birth is dexamethasone. Dexamethasone is a corticosteroid medication that is used to accelerate fetal lung maturation and reduce the risk of respiratory distress syndrome in premature infants. The drug is typically administered to pregnant women at risk of preterm labor between 24 and 34 weeks of gestation, and has been shown to be effective in reducing the incidence of respiratory distress syndrome and other neonatal complications. Misoprostol, oxytocin, and ginipral (ritodrine) are tocolytic medications that are used to suppress uterine contractions and delay preterm labor, but are not typically used to prevent fetal respiratory distress syndrome. Progesterone is a hormone that is involved in the maintenance of pregnancy, and may be prescribed to prevent preterm labor in some cases, but is not typically used to prevent fetal respiratory distress syndrome. Therefore, based on the information provided in the case, the most likely drug that would be prescribed to prevent fetal respiratory distress syndrome in a pregnant woman at risk of premature birth is dexamethasone. |
2. A 22-year-old pregnant woman was hospitalized in serious condition. Over the past three days, swelling, headache, nausea, and one-time vomiting appeared. Objectively observed: consciousness is dimmed, blood pressure — 160/130 mm Hg. art., small fibrillar twitching of the facial muscles, difficult nasal breathing. During transportation, the upper limbs began to twitch, the woman’s body stretched out, the spine was bent, the jaws were tightly clenched, and breathing stopped. After that, clonic convulsions and pronounced cyanosis appeared. Then the convulsions stopped, a deep, noisy breath appeared, foam stained with blood appeared on the lips. What is the most likely diagnosis?
a. Chorea
b. Hypertensive crisis
c. Eclampsia
d. Epilepsy
e. Diabetic coma
The correct answer is: Eclampsia
Explanation
The most likely diagnosis based on the symptoms and findings described in the case is eclampsia. Eclampsia is a serious complication of pregnancy characterized by seizures or convulsions in a woman with preeclampsia (high blood pressure and proteinuria) or gestational hypertension (high blood pressure without proteinuria) that cannot be attributed to other causes. The condition can lead to a range of symptoms, including headache, nausea, vomiting, visual disturbances, and altered mental status. In the case described, the woman presented with symptoms of preeclampsia, including swelling, headache, and hypertension. Her condition rapidly deteriorated, with the development of seizures and convulsions, difficult breathing, and cyanosis. These symptoms are consistent with eclampsia, a medical emergency that requires prompt evaluation and management by a obstetrician or other medical specialist. Chorea is a movement disorder characterized by involuntary, irregular movements of the limbs and face, and is not typically associated with pregnancy. Hypertensive crisis, epilepsy, and diabetic coma can cause similar symptoms to those described in the case, but are associated with different underlying causes and clinical presentations. Therefore, based on the information provided in the case, the most likely diagnosis is eclampsia, a serious complication of pregnancy that requires immediate medical attention to prevent further complications and ensure the safety of the mother and fetus. |
3. A 30-year-old woman complains of pain in the area of the heart (“pinches”, “drills”), which occurs mainly in the morning hours in the autumnspring period, with pain radiating to the neck, back, stomach; frequent heartbeat, as well as a decrease in general vitality. The occurrence of this condition is not related to physical activity. In the evening the condition improves. Somatic, neurological status and ECG – without pathology. What is the most likely pathology that caused such a clinical picture? Select one:
A. Somatized depression
B. Neurotic schizophrenia
C. Neurocirculatory dystonia
D. Hypochondriac depression
E. Angina at rest
The correct answer is: Somatized depression
Explanation
The most likely pathology that caused the clinical picture described in the case is somatized depression. Somatized depression is a type of depression in which psychological distress is expressed through physical symptoms. Common symptoms of somatized depression include pain, fatigue, gastrointestinal symptoms, and other somatic complaints that cannot be fully explained by a medical condition. The condition is often associated with anxiety and stress, and may be triggered by seasonal changes or other environmental factors. In the case described, the woman presents with symptoms of chest pain, neck pain, back pain, and stomach pain, as well as palpitations and decreased energy. These symptoms are not fully explained by a medical condition, as indicated by the normal somatic, neurological status, and ECG results. The symptoms are also associated with seasonal variation, suggesting a possible psychological or environmental trigger. Neurotic schizophrenia, neurocirculatory dystonia, hypochondriac depression, and angina at rest are other medical conditions that can cause similar symptoms to those described in the case, but are associated with different underlying causes, clinical presentations, and diagnostic criteria. Therefore, based on the information provided in the case, the most likely pathology that caused the clinical picture is somatized depression, a type of depression that is expressed through physical symptoms and is often associated with anxiety and stress. |
4. A 46-year-old patient first noticed swelling on his legs, weakness, a feeling of “fullness” and heaviness in the right hypochondrium. He has been suffering from rheumatoid arthritis for 20 years. Enlarged liver and spleen, dense consistency. Blood creatinine – 0.23 mmol/l, proteinemia – 68 g/l, cholesterol – 4.2 mmol/l, specific gravity of urine – 1012, proteinuria – 3.3 g/l, single wax-like cylinders, erythrocytes leached in p/ with, leukocytes – 5-6 in p/z. What is the most likely complication? Select one:
A. Kidney amyloidosis
B. Chronic glomerulonephritis
C. Chronic pyelonephritis
D. Heart failure
E. Acute glomerulonephritis
The correct answer is: Kidney amyloidosis
Explanation
The correct answer is A. Kidney amyloidosis. Kidney amyloidosis is a condition where abnormal protein called amyloid builds up in the kidneys, leading to damage and dysfunction of the organ. This condition is commonly associated with chronic inflammatory diseases, such as rheumatoid arthritis, and can cause a range of symptoms, including proteinuria, kidney failure, and swelling in the legs. The patient in this case has a history of rheumatoid arthritis and is presenting with symptoms that are consistent with kidney involvement. The presence of proteinuria, wax-like cylinders, and erythrocytes leached in p/ with, and leukocytes in p/z suggests that the kidneys are not functioning properly. Additionally, the dense consistency of the liver and spleen seen on physical examination is suggestive of amyloid deposition in these organs as well. Chronic glomerulonephritis and chronic pyelonephritis are also possible causes of kidney dysfunction, but in this case, the symptoms and clinical findings point more strongly towards kidney amyloidosis. Heart failure and acute glomerulonephritis are not the most likely complications in this case, as they are not commonly associated with long-standing rheumatoid arthritis. |
5. A 23-year-old patient complains of scanty menstruation, infertility, has gained 10 kg of weight over the past 6 months. Contraception methods are not used. When pressed from the mammary gland – milky secretions. The level of prolactin is elevated. During vaginal examination: the uterus is hypoplastic, the appendages are not palpable, their area is painless. Mucous discharge. What is the most likely diagnosis? Select one:
A. Hyperprolactinemia
B. Oligomenorrhea
C. Uterine hypoplasia
D. Breast cancer
E. Fibrocystic mastopathy
The correct answer is: Hyperprolactinemia
Explanation
The most likely diagnosis based on the symptoms and findings described in the case is hyperprolactinemia. Hyperprolactinemia is a medical condition characterized by elevated levels of prolactin, a hormone that stimulates milk production in the breast and regulates the menstrual cycle. The condition can be caused by a range of factors, including pituitary tumors, medications, and other underlying medical conditions. In the case described, the patient presents with symptoms of scanty menstruation, infertility, and milky secretions from the breast, which are indicative of hyperprolactinemia. Laboratory findings indicate elevated levels of prolactin, which further support this diagnosis. The patient also has gained weight over the past six months, which may be associated with hormonal imbalances. Uterine hypoplasia, oligomenorrhea, breast cancer, and fibrocystic mastopathy are other medical conditions that can cause similar symptoms to those described in the case, but are associated with different underlying causes, clinical presentations, and diagnostic criteria. Therefore, based on the information provided in the case, the most likely diagnosis is hyperprolactinemia, a medical condition characterized by elevated levels of prolactin that can cause a range of symptoms, including menstrual irregularities, infertility, and milk secretion from the breast. |
6. The patient complains of severe pain in the throat on the left side, difficulty swallowing and opening the mouth, increased body temperature, general malaise. She fell ill 4 days ago after angina. During the examination, the trismus of the masticatory muscles is determined, the left tonsil is shifted to the middle line, the anterior palatal arch is infiltrated and protrudes. Regional lymph nodes on the right are enlarged and painful on palpation. What is the diagnosis? Select one:
A. Lacunar angina
B. Paratonsillar abscess
C. Tonsil tumor
D. Infectious mononucleosis
E. Acute pharyngitis
The correct answer is: Peritonsillar abscess
Explanation
The most likely diagnosis based on the symptoms and findings described in the case is paratonsillar abscess. A peritonsillar abscess is a collection of pus that forms near the tonsils, usually as a complication of tonsillitis or other upper respiratory infections. The condition can cause severe pain in the throat, difficulty swallowing and opening the mouth, fever, and general malaise, among other symptoms. In the case described, the patient presents with symptoms of severe pain on the left side of the throat, difficulty swallowing and opening the mouth, increased body temperature, and general malaise, which are indicative of a peritonsillar abscess. Examination findings indicate trismus of the masticatory muscles, a shifted left tonsil, and an infiltrated and protruding anterior palatal arch, all of which are consistent with the diagnosis. Enlarged and painful regional lymph nodes on the right side further support the diagnosis. Lacunar angina, tonsil tumor, infectious mononucleosis, and acute pharyngitis are other medical conditions that can cause similar symptoms to those described in the case, but are associated with different underlying causes, clinical presentations, and diagnostic criteria. Therefore, based on the information provided in the case, the most likely diagnosis is paratonsillar abscess, a collection of pus near the tonsils that can cause severe pain in the throat, difficulty swallowing and opening the mouth, fever, and general malaise. |
7. The boy is 4 years old, vaccinated with a violation of the vaccination schedule. Complaints of sore throat when swallowing, headache, weakness, fever. Objectively: the child is pale, enlarged anterior cervical lymph nodes, swelling of the tonsils, their cyanotic hyperemia, gray-white plaque on the tonsils, does not come off, tonsils bleed when forcibly removed. What is the most likely diagnosis? Select one:
A. Infectious mononucleosis
B. Lacunar angina
C. Follicular angina
D. Oropharyngeal diphtheria
E. Simanovsky-Vincent’s angina
The correct answer is: Oropharyngeal diphtheria
Explanation
The most likely diagnosis based on the symptoms and findings described in the case is oropharyngeal diphtheria. Oropharyngeal diphtheria is an infectious disease caused by the bacterium Corynebacterium diphtheriae, which can cause a range of symptoms, including sore throat, fever, headache, weakness, and enlarged lymph nodes. The condition is characterized by the formation of a gray-white membrane or pseudomembrane on the tonsils or other parts of the throat, which can cause bleeding when forcibly removed. In the case described, the patient presents with symptoms of sore throat, headache, weakness, and fever, which are indicative of oropharyngeal diphtheria. Examination findings indicate pale skin, enlarged anterior cervical lymph nodes, swelling and cyanotic hyperemia of the tonsils, and the presence of gray-white plaque on the tonsils that does not come off easily and causes bleeding when forcibly removed. Infectious mononucleosis, lacunar angina, follicular angina, and Simanovsky-Vincent’s angina are other medical conditions that can cause similar symptoms to those described in the case, but are associated with different underlying causes, clinical presentations, and diagnostic criteria. Therefore, based on the information provided in the case, the most likely diagnosis is oropharyngeal diphtheria, an infectious disease caused by the bacterium Corynebacterium diphtheriae that can cause a range of symptoms, including sore throat, fever, headache, weakness, and the formation of a gray-white membrane or pseudomembrane on the tonsils or other parts of the throat. |
8. A 24-year-old patient turned to the doctor with complaints of an increase in submandibular lymph nodes. Objectively: enlarged submandibular, axillary and inguinal lymph nodes. An X-ray of the chest organs shows enlarged mediastinal lymph nodes. In the blood: er.- $3.4cdot10^{12$/l, Hb100 g/l, CP- 0.88, tr.- $190cdot10^9$/l, leuk.- $7.5 cdot10^9$/l, e.- 8%, p.- 2%, s.- 67%, lymph.- 23%, ESR- 22 mm/h. What research is indicated to verify the cause of lymphadenopathy? Select one:
A. Puncture biopsy of lymph nodes
B. Tomography of the mediastinum
C. Ultrasound examination of abdominal organs
D. Sternal puncture
E. Open biopsy of lymph nodes
The correct answer is: Open biopsy of lymph nodes
Explanation
The most appropriate research to verify the cause of lymphadenopathy in this case is a puncture biopsy of the lymph nodes. Lymphadenopathy is the medical term for the enlargement of lymph nodes, which can be caused by a range of factors, including infections, autoimmune disorders, and cancer. In this case, the patient presents with enlarged submandibular, axillary, inguinal, and mediastinal lymph nodes, as well as other laboratory findings that suggest abnormal blood cell counts. A puncture biopsy of the lymph nodes involves removing a small sample of tissue from the affected lymph node using a needle, which can then be examined under a microscope to identify the cause of the lymphadenopathy. This procedure is minimally invasive and can be performed on an outpatient basis. Tomography of the mediastinum, ultrasound examination of abdominal organs, sternal puncture, and open biopsy of lymph nodes are other diagnostic procedures that can be used to evaluate lymphadenopathy, but may be more invasive, expensive, or time-consuming than a puncture biopsy. Therefore, based on the information provided in the case, the most appropriate research to verify the cause of lymphadenopathy is a puncture biopsy of the lymph nodes. |
9. A 5-year-old child has been sick for 2 weeks. First there were coughing attacks, then – reruns. During coughing, the patient’s face turns red, neck veins swell. Coughing attacks end with vomiting. On the X-ray: strengthening of the bronchial pattern. Blood analysis: leukocytes – 16 10 /l, lymph – 72%, erythrocyte sedimentation rate – 4 mm/h. What is the most likely diagnosis? Select one:
A. Adenovirus infection
B. Foreign body
C. Pneumonia
D. Obstructive bronchitis
E. Whooping cough
The correct answer is: Кашлюĸ
Explanation
The most likely diagnosis based on the symptoms and findings described in the case is whooping cough. Whooping cough, also known as pertussis, is a highly contagious respiratory infection caused by the bacterium Bordetella pertussis. The condition is characterized by coughing attacks that are often severe and prolonged, and may be accompanied by a characteristic “whooping” sound when the patient inhales. In the case described, the patient presents with symptoms of coughing attacks that end with vomiting, facial redness, and swelling of the neck veins, which are indicative of whooping cough. Laboratory findings indicate an elevated white blood cell count with a high percentage of lymphocytes, which is also consistent with the diagnosis. X-ray findings suggest an increase in the bronchial pattern, which is a common finding in whooping cough. Adenovirus infection, foreign body, pneumonia, and obstructive bronchitis are other medical conditions that can cause similar symptoms to those described in the case, but are associated with different underlying causes, clinical presentations, and diagnostic criteria. Therefore, based on the information provided in the case, the most likely diagnosis is whooping cough, a highly contagious respiratory infection caused by the bacterium Bordetella pertussis that is characterized by severe and prolonged coughing attacks, facial redness, and swelling of the neck veins. |
10. As a result of the introduction of the new method, the average duration of treatment in the experimental group of patients was 12.3±0.2 days compared to 15.4±0.4 days in the control group of patients who were treated according to the old scheme. Which coefficient (criterion) can be calculated to assess the significance of the difference in these results? Select one:
A. Correspondence criterion (chi-square)
B. Sign criterion (z-criterion)
C. Wilcoxon’s t-test
D. Kolmogorov-Smirnov criterion
E. Odds ratio (Student’s t-test)
The correct answer is: Odds ratio (Student’s ttest)
Explanation
The most appropriate coefficient (criterion) to assess the significance of the difference in treatment duration between the experimental and control groups is Student’s t-test. Student’s t-test is a statistical test used to determine whether there is a significant difference between the means of two independent groups. It is commonly used in medical research to compare the effectiveness of different treatments or interventions. The test calculates a t-value, which represents the difference between the means of the two groups divided by the standard error of the difference. In the case described, the experimental group of patients was treated with a new method that resulted in an average treatment duration of 12.3±0.2 days, while the control group of patients was treated with an old scheme that resulted in an average treatment duration of 15.4±0.4 days. Student’s t-test can be used to calculate the significance of this difference in treatment duration between the two groups. Correspondence criterion (chi-square), sign criterion (z-criterion), Wilcoxon’s t-test, and Kolmogorov-Smirnov criterion are other statistical tests that can be used to evaluate differences between two groups, but they are better suited for different types of data or research questions. Therefore, based on the information provided in the case, the most appropriate coefficient (criterion) to assess the significance of the difference in treatment duration between the experimental and control groups is Student’s t-test. |
11. The incidence of cervical cancer is increasing in the rural medical district. A decision was made to conduct an examination of women. What kind of medical examinations are these? Select one:
A. Screening
B. Previous
C. targeted
D. Current
E. Complex
The correct answer is: Targeted
Explanation
The most appropriate medical examination to address the increasing incidence of cervical cancer in a rural medical district is a targeted examination. A targeted examination is a medical examination that is focused on a specific population, based on risk factors or other criteria. In this case, the decision to conduct an examination of women is based on the increasing incidence of cervical cancer in the rural medical district, which suggests that women in this population may be at higher risk for the condition. Screening examinations are another type of medical examination that is commonly used to identify individuals at risk for a particular condition, but they are typically applied to larger populations and may involve broader criteria for selection. Previous, current, and complex examinations are not specific types of medical examinations, and their meaning may depend on the context in which they are used. Therefore, based on the information provided in the case, the most appropriate medical examination to address the increasing incidence of cervical cancer in a rural medical district is a targeted examination, which is focused on women who may be at higher risk for the condition. |
12. A 39-year-old patient complains of shortness of breath and palpitations during physical exertion. From the mother’s words: in childhood, heart murmurs were listened to, but no examinations were performed. Objectively: the skin is pale, the pulse is 94/min, rhythmic. Blood pressure – 120/60 mm Hg. A coarse systolic-diastolic murmur is heard in the II intercostal space on the left, the accent of the II tone over the pulmonary artery. General analysis of blood and urine without changes. What is the most likely diagnosis for the patient? Select one:
A. Tetralogy of Fallot
B. Coarctation of the aorta
C. The ductus arteriosus is open
D. Atrial septal defect
E. Ventricular septal defect
The correct answer is: Open ductus arteriosus
Explanation
The most likely diagnosis based on the symptoms and findings described in the case is an open ductus arteriosus. An open ductus arteriosus is a congenital heart defect characterized by the failure of the ductus arteriosus, a blood vessel that connects the pulmonary artery to the aorta in the fetus, to close after birth. The condition can cause symptoms such as shortness of breath, palpitations, and heart murmurs, and can lead to complications such as heart failure and pulmonary hypertension. In the case described, the patient presents with symptoms of shortness of breath and palpitations during physical exertion, as well as a history of heart murmurs in childhood. Examination findings indicate a coarse systolic-diastolic murmur in the II intercostal space on the left and an accent of the II tone over the pulmonary artery, which are consistent with an open ductus arteriosus. Laboratory findings are within normal limits. Tetralogy of Fallot, coarctation of the aorta, atrial septal defect, and ventricular septal defect are other congenital heart defects that can cause similar symptoms and findings, but are associated with different underlying anatomical abnormalities and clinical presentations. Therefore, based on the information provided in the case, the most likely diagnosis is an open ductus arteriosus, a congenital heart defect characterized by the failure of the ductus arteriosus to close after birth, which can cause symptoms such as shortness of breath, palpitations, and heart murmurs, and can lead to complications such as heart failure and pulmonary hypertension. |
13. A 58-year-old man came to the clinic due to an attack of renal colic, which periodically recurs during the year. Objectively: in the area of the auricles and the right elbow joint there are nodular formations covered with thin shiny skin. Ps88/min., BP- 170/100 mm Hg. Positive Pasternacki symptom on both sides. The patient is scheduled for an examination. The study of which laboratory indicator is most appropriate for establishing a diagnosis? Select one:
A. Urine sediment
B. Lactic acid
C. SOE
D. Rheumatoid factor
E. Uric acid
The correct answer is: Uric acid
Explanation
The most appropriate laboratory indicator to establish a diagnosis in this case is uric acid. Uric acid is a waste product that is produced by the body during the breakdown of purines, which are found in many foods and are also produced by the body. High levels of uric acid in the blood can lead to the formation of urate crystals, which can accumulate in the joints and cause gout, a type of inflammatory arthritis that is characterized by sudden and severe attacks of pain, redness, and swelling in the affected joint. In the case described, the patient presents with a history of recurrent renal colic, as well as nodular formations in the area of the auricles and the right elbow joint, which are suggestive of gout. Examination findings indicate an elevated pulse rate and blood pressure, as well as a positive Pasternacki symptom on both sides, which are also consistent with gout. Laboratory testing can confirm the diagnosis by measuring the level of uric acid in the blood. Urine sediment, lactic acid, SOE (erythrocyte sedimentation rate), and rheumatoid factor are other laboratory indicators that can be useful in the diagnosis of various medical conditions, but are less specific for gout than uric acid. Therefore, based on the information provided in the case, the most appropriate laboratory indicator to establish a diagnosis is uric acid, which can confirm the presence of high levels of uric acid in the blood and support a diagnosis of gout. |
14. In a pregnant woman suffering from hypertension of the first degree, in the period of 35 weeks, edema appeared on the lower extremities and the anterior abdominal wall, the amount of protein in the daily urine — up to 5 g/l, blood pressure increased to 170/120 mm Hg .st., a headache began to bother him and his vision worsened. Intensive treatment carried out for four hours had no effect. What tactics are necessary in this situation?
a. Continuation of intensive therapy
b. Preparation of the cervix for premature birth
c. Immediate delivery by caesarean section
d. Carrying out induction of labor
e. Conservative birth
The correct answer is: Immediate delivery by caesarean section
Explanation
15. The corpse of a citizen was found at his residence. On the face, neck and hands, areas of irregular shape were found, measuring from 2 x 3 cm to 4 x 5 cm. The skin and underlying tissues are absent in these areas. The edges of the damage are uneven, large and small scalloped, without signs of bleeding. What is the mechanism of these injuries? Select one:
A. Fire damage (shotgun)
B. Numerous stab wounds
C. Damage caused by animals
D. Blast injury
E. Local effect of cold
The correct answer is: Damage caused by animals
Explanation
The mechanism of the injuries described in the case is damage caused by animals. The irregular shape of the areas, the absence of skin and underlying tissues, and the uneven, scalloped edges without signs of bleeding are all characteristic of animal bites or scratches. The size and shape of the areas suggest that the injuries were caused by a relatively large animal, such as a dog or a large wild animal. Firearm injuries, stab wounds, blast injuries, and local effects of cold are all potential mechanisms of injury, but they are less likely to produce the specific pattern of injuries described in the case. Therefore, based on the information provided in the case, the most likely mechanism of the injuries is damage caused by animals, such as bites or scratches from a large animal. |
16. A woman came to the gynecologist on the 20th day of the postpartum period with complaints of pain in the left mammary gland, purulent discharge from the nipple. Objectively: Ps120/min., t° of the body – 39 C. The left mammary gland is painful, larger than the right, its skin is hyperemic, in the upper quadrant – an infiltrate of 10 x 15 cm with softening inside. In the blood: ESR – 50 mm/hour, leuk. – 15.0 • 10 /l. What will be the doctor’s tactics? Select one:
A. Send to the postpartum department
B. Hospitalize to the surgical department for operative treatment
C. Send to the gynecological department
D. Send to a polyclinic surgeon for conservative treatment
E. Open a mammary gland abscess in a women’s consultation o 9
The correct answer is: Hospitalize to the surgical department for operative treatment
Explanation
The most appropriate doctor’s tactics in this case is to hospitalize the patient to the surgical department for operative treatment. The symptoms and findings described in the case are suggestive of a breast abscess, which is a localized collection of pus within the breast tissue that can occur in the postpartum period due to infection or inflammation of the breast tissue. The presence of fever, leukocytosis, and a large, painful infiltrate with softening inside are all consistent with a breast abscess. Without prompt treatment, a breast abscess can lead to serious complications, such as sepsis or breast tissue necrosis. In this case, hospitalization to the surgical department for operative treatment, such as incision and drainage of the abscess, is necessary to prevent further complications and promote healing. Conservative treatment, such as antibiotics or aspiration of the abscess, may not be sufficient to address the underlying infection and may delay healing. Sending the patient to the postpartum department or gynecological department may not be appropriate, as the patient’s condition requires surgical intervention. Sending the patient to a polyclinic surgeon for conservative treatment may also not be appropriate, as the patient’s symptoms and findings suggest a large and potentially complicated abscess that requires prompt surgical management. Therefore, based on the information provided in the case, the most appropriate doctor’s tactics is to hospitalize the patient to the surgical department for operative treatment, such as incision and drainage of the breast abscess. |
17. A 24-year-old pregnant woman came to the obstetric hospital at 37 weeks of pregnancy with complaints of weak fetal movements. Fetal heartbeat – 95/min. During the vaginal examination, the cervix is turned back, 2 cm long, the external eye passes the tip of the finger. The biophysical profile of the fetus is 4 points. What tactic of managing a pregnant woman should be chosen? Select one:
A. Conduct Doppler blood flow velocity measurement in the umbilical artery
B. Prescribe immediate preparation of the cervix for childbirth
C. Treat placental dysfunction and redetermine the biophysical profile of the fetus the next day
D. Emergency delivery by caesarean section
E. Carry out treatment of fetal distress, if there is no effect, carry out a cesarean section as planned the next day
The correct answer is: Emergency delivery by caesarean section
Explanation
The most appropriate tactic of managing a pregnant woman in this case is emergency delivery by caesarean section. The symptoms and findings described in the case are suggestive of fetal distress, which is a serious condition in which the fetus is not receiving enough oxygen and nutrients, and can lead to fetal death or permanent brain damage if left untreated. The low fetal heart rate, weak fetal movements, and low biophysical profile score all suggest fetal distress. The presence of a posterior cervix and a partially effaced external os may indicate fetal head compression, which can further compromise fetal oxygenation. In this case, prompt delivery by caesarean section is necessary to prevent further fetal distress and improve fetal outcomes. Doppler blood flow velocity measurement in the umbilical artery may provide additional information about fetal well-being, but it may not be sufficient to address the underlying cause of fetal distress and may delay delivery. Preparing the cervix for childbirth or treating placental dysfunction may also not be appropriate, as the patient’s condition requires immediate intervention to prevent further complications. Therefore, based on the information provided in the case, the most appropriate tactic of managing a pregnant woman is emergency delivery by caesarean section, which is necessary to prevent further fetal distress and improve fetal outcomes. |
18. A 45-year-old patient has been suffering from ankylosing spondylitis for 15 years. For the past three years, he has noted pastiness of the face and swelling of the limbs. Objectively: the pose of a “supplier”. Radiologically: changes in the thoracic and lumbar parts of the spine according to the “bamboo stick” type. Ultrasound of the heart: aortic regurgitation. ZAC: Hb – 106 g/l; L – 8.9 G/l; ESR – 40 mm/h. Daily proteinuria – 9.6 g/d. Blood creatinine – 230 µmol/l. What is the cause of the development of kidney failure? Select one:
A. Pyelonephritis
B. Concomitant pathology of the heart
C. Side effect of drugs
D. Kidney amyloidosis
E. Urinary stone disease
The correct answer is: Kidney amyloidosis
Explanation
The cause of the development of kidney failure in this case is most likely kidney amyloidosis. Ankylosing spondylitis is a chronic inflammatory disease that can affect multiple organs, including the kidneys. Amyloidosis is a complication of ankylosing spondylitis that can lead to kidney failure. In amyloidosis, abnormal proteins called amyloids build up in various organs, including the kidneys, and interfere with their normal function. The presence of proteinuria, elevated creatinine, and anemia in the case suggest kidney involvement. The aortic regurgitation seen on ultrasound of the heart is likely a result of amyloid deposition in the heart valves and is also a common complication of amyloidosis. Pyelonephritis, urinary stone disease, and concomitant pathology of the heart may contribute to kidney dysfunction, but they are less likely to cause kidney failure in a patient with ankylosing spondylitis and a history of amyloidosis. Side effects of drugs may also contribute to kidney dysfunction, but it is unlikely to cause kidney failure in the absence of other contributing factors. Therefore, based on the information provided in the case, the most likely cause of the development of kidney failure is kidney amyloidosis, a complication of ankylosing spondylitis that can lead to the deposition of abnormal proteins in the kidneys and interfere with their normal function. |
19. The employee was on a business trip in another city, where he fell ill and was hospitalized in a city hospital. How should he be issued a certificate of incapacity for work? Select one:
A. With the permission of the chief physician of the city hospital
B. With the permission of the deputy chief physician for medical work
C. The attending physician and the head of the hospital department
D. With the permission of the deputy chief physician for a disability examination
E. Treating doctor
The correct answer is: With the permission of the chief physician of the city hospital
Explanation
The most appropriate way to issue a certificate of incapacity for work for an employee who was hospitalized in a city hospital while on a business trip is with the permission of the chief physician of the city hospital. The chief physician of the hospital is the highest-ranking medical officer responsible for the overall management and administration of the hospital. As such, they have the authority to issue a certificate of incapacity for work for a patient who is hospitalized in the hospital under their care. The attending physician and the head of the hospital department may also be involved in the process of issuing the certificate, but the final decision is typically made by the chief physician. The deputy chief physician for medical work and the deputy chief physician for disability examination may also play a role in the process of issuing a certificate of incapacity for work, but their involvement may depend on the specific policies and procedures of the hospital. Therefore, based on the information provided in the case, the most appropriate way to issue a certificate of incapacity for work for the employee who was hospitalized in a city hospital while on a business trip is with the permission of the chief physician of the city hospital. |
20. What modern organizational method makes it possible to provide patients with timely access to medical care of appropriate quality, to receive a medical service for counseling, diagnosis and treatment in remote settlements, especially in cases where distance and time are critical factors in providing medical care? Select one:
A. Mobile Communication
B. Ambulance
C. Sanaviation
D. Telemedicine
E. Field teams of specialists
The correct answer is: Telemedicine
Explanation
The modern organizational method that makes it possible to provide patients with timely access to medical care of appropriate quality, to receive a medical service for counseling, diagnosis and treatment in remote settlements, especially in cases where distance and time are critical factors in providing medical care is telemedicine. Telemedicine refers to the use of telecommunication and information technologies to provide healthcare services remotely. It allows healthcare providers to consult with patients, diagnose medical conditions, and provide treatment without the need for physical presence. Telemedicine can be especially useful in remote or underserved areas where access to medical care is limited. It can also be used to provide medical services in emergencies or critical situations where time and distance are critical factors in providing medical care. Other options such as mobile communication, ambulance, sanaviation, and field teams of specialists may also play a role in providing medical care in remote areas, but they do not offer the same level of flexibility and access to medical expertise as telemedicine. Therefore, based on the information provided in the question, the most appropriate modern organizational method for providing medical care in remote settlements is telemedicine. |
21. The patient is 29 years old, an auto mechanic, with a history of frequent hypothermia, exacerbation of chronic bronchitis, which is accompanied by a cough with relatively scanty mucous-purulent sputum, low-grade fever, sometimes hemoptysis and pain in the right half of the chest. Breathing is vesicular. Radiologically, darkening and a sharp decrease in the volume of the lower lobe are determined, which is clearly visible on the X-ray in the form of a strip 2-3 cm wide, which goes obliquely from the root of the lung to the anterior costo-diaphragmatic sinus. What is the most likely diagnosis? Select one:
A. Middle lobe syndrome
B. Interlobular pleurisy
C. Peripheral lung cancer
D. Pneumonia
E. Bronchiectasis
The correct answer is: Peripheral lung cancer
Explanation
The most likely diagnosis in this case is peripheral lung cancer. The patient’s history of chronic bronchitis and frequent hypothermia may increase the risk of developing lung cancer, particularly in the setting of hemoptysis and chest pain. The presence of a cough with mucous-purulent sputum, low-grade fever, and a clear radiological finding of a strip-like opacity in the lower lobe of the lung are also suggestive of lung cancer. While other conditions such as pneumonia, bronchiectasis, middle lobe syndrome, and interlobular pleurisy may also be considered in the differential diagnosis, the presence of a clear radiological finding of a strip-like opacity in the lower lobe of the lung, along with the patient’s history and symptoms, make peripheral lung cancer the most likely diagnosis. Therefore, based on the information provided in the case, the most likely diagnosis is peripheral lung cancer, and further diagnostic tests, such as a biopsy or CT scan, should be performed to confirm the diagnosis and determine the extent of the disease. |
22. As a result of the explosion of a tank with benzene at a chemical plant, there were a large number of dead and injured (more than 50 people) with burns, mechanical injuries and poisoning. Specify the main elements that provide medical and evacuation support for the population in this situation: Select one:
A. Sorting, providing medical assistance, evacuation
B. Sorting, evacuation, treatment
C. Sorting, recovery, rescue
D. Isolation, rescue, recovery
E. Provision of medical assistance, evacuation, isolation
The correct answer is: Sorting, providing medical assistance, evacuation
Explanation
The main elements that provide medical and evacuation support for the population in the situation described are sorting, providing medical assistance, and evacuation. Sorting involves the initial assessment and classification of patients based on the severity of their injuries and the urgency of their need for medical care. This allows medical personnel to prioritize treatment and evacuation efforts and allocate resources accordingly. In a mass casualty incident such as an explosion, sorting is critical for effective triage and management of patients. Providing medical assistance involves the provision of immediate, on-site medical care to stabilize patients and prevent further deterioration of their condition. This may include basic life support, wound care, and administration of medications to manage pain, reduce inflammation, or address other medical issues. Evacuation involves the timely and safe transport of patients to appropriate medical facilities for further treatment and care. In a mass casualty incident, evacuation may be complicated by the large number of patients and the severity of their injuries. Other elements such as recovery, rescue, isolation, and treatment may also be important in certain situations, but sorting, providing medical assistance, and evacuation are generally considered the key elements of medical and evacuation support in a mass casualty incident such as the one described. Therefore, based on the information provided in the question, the main elements that provide medical and evacuation support for the population in the situation described are sorting, providing medical assistance, and evacuation. |
23. A 10-year-old boy suffered viral hepatitis B 4 years ago. At present, it is assumed that cirrhosis of the liver has developed. What additional examination will help establish the diagnosis? Select one:
A. Echocholecystography
B. Determination of hepatitis B markers
C. Proteinogram
D. Determination of the level of transaminases
E. Puncture biopsy of the liver
The correct answer is: Liver puncture biopsy
Explanation
The additional examination that will help establish the diagnosis of cirrhosis of the liver in a 10-year-old boy who suffered viral hepatitis B 4 years ago is a liver puncture biopsy. Cirrhosis of the liver is a chronic liver disease characterized by fibrosis and scarring of the liver tissue. It can be caused by various factors, including viral hepatitis B. The diagnosis of cirrhosis is usually based on a combination of clinical, laboratory, and imaging findings. However, in some cases, a liver puncture biopsy may be necessary to confirm the diagnosis and assess the extent of liver damage. Liver puncture biopsy involves the insertion of a needle through the skin and into the liver to obtain a small sample of liver tissue for examination under a microscope. This procedure is typically performed under local anesthesia and with imaging guidance to ensure accuracy and safety. Other tests such as determination of hepatitis B markers, proteinogram, and determination of the level of transaminases may provide additional information about liver function and the underlying cause of liver damage, but they may not be sufficient to establish the diagnosis of cirrhosis. Therefore, based on the information provided in the question, the additional examination that will help establish the diagnosis of cirrhosis of the liver in a 10-year-old boy who suffered viral hepatitis B 4 years ago is a liver puncture biopsy. |
24. A 43-year-old patient, a miner-slaughterer with 15 years of work experience, complains of cough, chest pain, shortness of breath. The cough is slight, mostly in the morning, usually dry. The pain is localized in the interscapular region, intensifies with deep inhalation. Shortness of breath during physical exertion. In the lungs – weakened vesicular breathing. Heart sounds – rhythmic, heart rate – 86/min., blood pressure – 135/80 mm Hg. Abdomen is soft, painless. X-ray: small nodular pulmonary fibrosis. What is the previous diagnosis? Select one:
A. Carboconiosis
B. Beryliosis
C. Siderosis
D. Bisinosis
E. Metalloconiosis
The correct answer is: Carboconiosis
Explanation
The previous diagnosis for the 43-year-old patient who is a miner-slaughterer with 15 years of work experience, complaining of cough, chest pain, shortness of breath, and having small nodular pulmonary fibrosis on X-ray, is likely to be carboconiosis. Carboconiosis, also known as coal worker’s pneumoconiosis (CWP), is a type of occupational lung disease caused by long-term inhalation of coal dust. It is commonly seen in miners, such as the patient in this case. The symptoms of carboconiosis include cough, chest pain, shortness of breath, and weakened vesicular breathing. The small nodular pulmonary fibrosis seen on X-ray is a characteristic finding in carboconiosis. Beryliosis, siderosis, bisinosis, and metalloconiosis are other types of occupational lung diseases, but they are caused by exposure to other types of dust or metal particles, and are less likely to be the cause of the patient’s symptoms and X-ray findings. Therefore, based on the information provided in the case, the most likely previous diagnosis for the 43-year-old patient who is a miner-slaughterer with 15 years of work experience, complaining of cough, chest pain, shortness of breath, and having small nodular pulmonary fibrosis on X-ray, is carboconiosis. |
25. A 14-year-old girl lost consciousness during the meeting, she complained of a headache the day before. The skin is pale, the extremities are cold, the breathing is shallow, the heart sounds are weakened, the heart rate is 51/min., the blood pressure is 90/50 mm Hg. The stomach is soft. Meningeal symptoms are negative. What is the previous diagnosis? Select one:
A. Acute left ventricular failure
B. Collapse
C. Acute right ventricular failure
D. Faint
E. Respiratory failure
The correct answer is: Collapse
Explanation
The most likely previous diagnosis for the 14-year-old girl who lost consciousness during a meeting, complained of a headache the day before, has pale skin, cold extremities, shallow breathing, weakened heart sounds, heart rate of 51/min, and blood pressure of 90/50 mm Hg, with negative meningeal symptoms, is collapse. Collapse is a sudden and temporary loss of consciousness and posture, with spontaneous recovery, caused by insufficient blood flow to the brain. Common causes of collapse include dehydration, low blood sugar, and cardiovascular problems such as arrhythmias or hypotension. In this case, the girl’s symptoms, including pale skin, cold extremities, shallow breathing, weakened heart sounds, low heart rate, and low blood pressure, suggest a decreased cardiac output, which can lead to inadequate blood flow to the brain and result in collapse. Acute left ventricular failure, acute right ventricular failure, respiratory failure, and fainting are also potential causes of collapse, but the information provided in the case does not suggest a specific underlying condition that would support any of these diagnoses. Therefore, based on the information provided in the case, the most likely previous diagnosis for the 14-year-old girl who lost consciousness during a meeting and has the described symptoms is collapse, and further diagnostic tests may be necessary to identify the underlying cause. |
26. During the medical examination of middle and high school students, doctors determined the correspondence of biological development and calendar age according to the following criteria: annual increase in body length, ossification of hand bones, number of permanent teeth. What additional developmental indicator should doctors most likely include in these age periods? Select one:
A. The vital capacity of the lungs
B. Circumference of the chest
C. Muscular strength of the hand
D. Body weight
E. Development of secondary sexual characteristics
The correct answer is: Development of secondary sexual characteristics
Explanation
The additional developmental indicator that doctors should most likely include in the age periods of middle and high school students, alongside the criteria of annual increase in body length, ossification of hand bones, and number of permanent teeth, is the development of secondary sexual characteristics. During puberty, which typically occurs during middle and high school years, adolescents undergo significant physical changes, including the development of secondary sexual characteristics such as breast development in girls and pubic hair growth and voice deepening in boys. These changes are important indicators of biological development and can provide valuable information about an adolescent’s growth and development. The vital capacity of the lungs, circumference of the chest, muscular strength of the hand, and body weight are also important indicators of physical development, but they may not be as closely linked to age and puberty as the development of secondary sexual characteristics. Therefore, based on the information provided in the question, the additional developmental indicator that doctors should most likely include in the age periods of middle and high school students, alongside the criteria of annual increase in body length, ossification of hand bones, and number of permanent teeth, is the development of secondary sexual characteristics. |
27. A 37-year-old patient is being treated for a month for discogenic lumbosacral sciatica. Numbness of the skin appears on the lateral side of the right lower limb, the Achilles reflex is absent. During the MRI examination of the lumbar spine – prolapse of the intervertebral disc $L_5$-$S_1$ up to 8 mm. Name the further tactics of treating the patient: Select one:
A. Stabilizing surgery on the spine
B. Operative removal of the intervertebral disc
C. Manual therapy
D. Conservative treatment
E. Using a corset
The correct answer is: Operative removal of the intervertebral disc
Explanation
The further treatment tactics for the 37-year-old patient who has been treated for discogenic lumbosacral sciatica for a month, with numbness of the skin on the lateral side of the right lower limb and absent Achilles reflex, and an MRI showing prolapse of the intervertebral disc $L_5$-$S_1$ up to 8 mm, is operative removal of the intervertebral disc. Intervertebral disc prolapse occurs when the soft inner portion of the disc protrudes through the outer layer and presses on nearby nerves, causing pain, numbness, and weakness in the affected area. In cases of severe or progressive symptoms, surgical intervention may be necessary to relieve pressure on the affected nerve. Operative removal of the intervertebral disc, also known as discectomy, involves the removal of the damaged or protruding portion of the disc to relieve pressure on the affected nerve and reduce symptoms. This can be performed using minimally invasive techniques or traditional open surgery, depending on the specific case. Conservative treatment, such as rest, physical therapy, and medication, may be effective for mild or moderate cases of discogenic lumbosacral sciatica, but is unlikely to be effective in cases with severe or progressive symptoms such as numbness and absent reflexes. Manual therapy and the use of a corset may provide temporary relief of symptoms, but are not likely to address the underlying cause of the problem and may not be sufficient to prevent the progression of the condition. Therefore, based on the information provided in the question, the further treatment tactics for the 37-year-old patient with discogenic lumbosacral sciatica, numbness of the skin on the lateral side of the right lower limb, and absent Achilles reflex, with an MRI showing prolapse of the intervertebral disc $L_5$-$S_1$ up to 8 mm, is operative removal of the intervertebral disc. |
28. A 72-year-old patient with a diagnosis of coronary heart disease (CHD), diffuse cardiosclerosis, atrial fibrillation, permanent form, tachysystolic, HF IIa, FC III. according to objective research data: blood pressure – 135/80 mm Hg, heart rate – 162/min, pulse – 126/min. Left ventricular ejection fraction – 32%. Which of the following medicines has indications for appointment in this case? Select one:
A. Verapamil
B. Ivabradin
C. Novocainamide
D. Izadrin
E. Digoxin
The correct answer is: Дігоĸсин
Explanation
The medicine that has indications for appointment in the case of a 72-year-old patient with a diagnosis of coronary heart disease (CHD), diffuse cardiosclerosis, atrial fibrillation, permanent form, tachysystolic, HF IIa, FC III, blood pressure of 135/80 mm Hg, heart rate of 162/min, pulse of 126/min, and left ventricular ejection fraction of 32%, is digoxin. Digoxin is a medication commonly used to treat heart failure and atrial fibrillation. It works by slowing down the heart rate and increasing the strength of the heart’s contractions, which can help improve symptoms and reduce the risk of complications. In this case, the patient has atrial fibrillation with a high heart rate of 162/min, which can increase the risk of complications such as stroke and heart failure. Digoxin can help control the heart rate and improve symptoms in patients with atrial fibrillation and heart failure, and is therefore an appropriate medication for this patient. Verapamil and ivabradine are also medications used to control heart rate in patients with atrial fibrillation, but they may not be suitable for patients with heart failure or low ejection fraction. Novocainamide and izadrin are not commonly used to treat atrial fibrillation or heart failure, and may have significant side effects and contraindications. Therefore, based on the information provided in the question, the medicine that has indications for appointment in the case of a 72-year-old patient with a diagnosis of coronary heart disease, diffuse cardiosclerosis, atrial fibrillation, permanent form, tachysystolic, HF IIa, FC III, and left ventricular ejection fraction of 32%, is digoxin. |
29. A 7-year-old child became acutely ill. He complains of a headache. There was vomiting twice. Objectively: inhibited, body temperature 39.3 C, pronounced hyperesthesia, stiffness of the occipital muscles, positive symptoms of upper and lower Brudzinsky, symmetrical Kernig. What is the previous diagnosis? Select one:
A. Toxic encephalopathy
B. Food poisoning
C. Meningitis
D. Encephalitis
E. Brain injury o
The correct answer is: Meningitis
Explanation
The most likely previous diagnosis for the 7-year-old child who became acutely ill, complains of a headache, vomited twice, is inhibited, has a body temperature of 39.3 C, pronounced hyperesthesia, stiffness of the occipital muscles, and positive symptoms of upper and lower Brudzinsky and symmetrical Kernig, is meningitis. Meningitis is an inflammation of the membranes (meninges) surrounding the brain and spinal cord, typically caused by a bacterial or viral infection. Symptoms of meningitis in children may include headache, vomiting, fever, hyperesthesia, and stiffness of the neck and back muscles. Positive symptoms of upper and lower Brudzinsky and symmetrical Kernig are also characteristic signs of meningitis. Toxic encephalopathy, food poisoning, encephalitis, and brain injury are other possible causes of headache, vomiting, and fever in children, but these conditions are less likely to cause the specific signs of meningitis observed in this case. Therefore, based on the information provided in the question, the most likely previous diagnosis for the 7-year-old child who became acutely ill, complains of a headache, vomited twice, is inhibited, has a body temperature of 39.3 C, pronounced hyperesthesia, stiffness of the occipital muscles, and positive symptoms of upper and lower Brudzinsky and symmetrical Kernig, is meningitis. Urgent medical attention is needed to confirm the diagnosis and initiate appropriate treatment. |
30. A 23-year-old man complains of facial swelling, headache, dizziness, decreased urine output, change in urine color (dark red). The above complaints appeared after the acute tonsillitis. Objectively: facial swelling, pale skin, temperature – 37.4°C; heart rate – 86/min., blood pressure – 170/110 mm Hg. The heart tones are muffled, the accent of the second tone over the aorta. What is the most likely etiological factor for this disease? Select one:
A. Staphylococcus aureus
B. β-hemolytic streptococcus
C. Green Streptococcus
D. Saprophytic staphylococcus
E. Streptococcus pyogenes
The correct answer is: β-hemolytic streptococcus
Explanation
The most likely etiological factor for the disease of the 23-year-old man who complains of facial swelling, headache, dizziness, decreased urine output, and change in urine color (dark red) after acute tonsillitis, with facial swelling, pale skin, temperature of 37.4°C, heart rate of 86/min., blood pressure of 170/110 mm Hg, muffled heart tones, and accent of the second tone over the aorta, is β-hemolytic streptococcus. Acute glomerulonephritis is a kidney disease that can occur as a complication of a streptococcal infection, such as acute tonsillitis. The symptoms of acute glomerulonephritis may include facial swelling, headache, dizziness, decreased urine output, and change in urine color, as well as high blood pressure, muffled heart tones, and accent of the second tone over the aorta. β-hemolytic streptococcus is a common cause of acute tonsillitis and can lead to complications such as acute glomerulonephritis. Other streptococcal strains, such as Streptococcus pyogenes, may also cause acute tonsillitis and subsequent acute glomerulonephritis. Staphylococcus aureus, Green Streptococcus, and Saprophytic staphylococcus are not typically associated with acute tonsillitis or acute glomerulonephritis. Therefore, based on the information provided in the question, the most likely etiological factor for the disease of the 23-year-old man who complains of facial swelling, headache, dizziness, decreased urine output, and change in urine color after acute tonsillitis, with facial swelling, pale skin, temperature of 37.4°C, heart rate of 86/min., blood pressure of 170/110 mm Hg, muffled heart tones, and accent of the second tone over the aorta, is β-hemolytic streptococcus. |
31. On the 14th day after giving birth, a woman in labor turned to the doctor with complaints of sudden pain, hyperemia and thickening in the mammary gland on the left, temperature rise to 39 o C headache, malaise. Objectively: a crack in the nipple area, an increase in the volume of the left mammary gland, increased pain during its palpation. What pathology can be thought of in this case? Select one:
A. Left mammary gland cyst with suppuration
B. Phlegmon of the mammary gland
C. Breast cancer
D. Fibroadenoma of the left mammary gland
E. Lactation mastitis ,
The correct answer is: Lactational mastitis
Explanation
The pathology that can be thought of in the case of a woman on the 14th day after giving birth, with sudden pain, hyperemia, and thickening in the mammary gland on the left, a temperature rise to 39°C, headache, malaise, a crack in the nipple area, an increase in the volume of the left mammary gland, and increased pain during its palpation, is lactational mastitis. Lactational mastitis is an inflammation of the breast tissue that can occur in breastfeeding women due to milk stasis, bacterial infection, or both. Symptoms of lactational mastitis may include sudden onset of pain, redness, and swelling in the breast, fever, and flu-like symptoms. A crack in the nipple area can also increase the risk of infection and subsequent mastitis. Left mammary gland cyst with suppuration, phlegmon of the mammary gland, breast cancer, and fibroadenoma of the left mammary gland are less likely to cause the specific symptoms observed in this case, especially in the context of recent childbirth and breastfeeding. Therefore, based on the information provided in the question, the pathology that can be thought of in the case of a woman on the 14th day after giving birth, with sudden pain, hyperemia, and thickening in the mammary gland on the left, a temperature rise to 39°C, headache, malaise, a crack in the nipple area, an increase in the volume of the left mammary gland, and increased pain during its palpation, is lactational mastitis. Urgent medical attention is needed to confirm the diagnosis and initiate appropriate treatment, which may include antibiotics and measures to improve milk flow and drainage from the affected breast. |
32. A 58-year-old patient complains of pain in the left lower extremity, which worsens when walking, a feeling of cooling and numbness in both feet. He has been sick for 6 years. Objectively: the skin is pale, dry; hyperkeratosis is observed. On the left lower leg, the hair cover is poorly developed, the “furrow” symptom is positive. The pulse on the arteries of the foot, popliteal artery is not determined, on the femoral artery – weakened. On the right limb, the pulsation of the popliteal artery is preserved. What is the most likely diagnosis? Select one:
A. Buerger’s disease
B. Thrombosis of the femoral artery
C. Obliterating atherosclerosis of the arteries of the lower extremities
D. Raynaud’s disease
E. Obliterating endarteritis
The correct answer is: Obliterating atherosclerosis of the arteries of the lower extremities
Explanation
The most likely diagnosis for the 58-year-old patient who complains of pain in the left lower extremity, which worsens when walking, a feeling of cooling and numbness in both feet, with pale, dry skin, hyperkeratosis, poorly developed hair cover and positive “furrow” symptom on the left lower leg, and absence of pulse on the arteries of the foot and popliteal artery and weakened pulse on the femoral artery, is obliterating atherosclerosis of the arteries of the lower extremities. Obliterating atherosclerosis of the arteries of the lower extremities is a progressive narrowing and blockage of the arteries that supply blood to the legs and feet. Symptoms of obliterating atherosclerosis may include pain, numbness, and cooling sensations in the affected limb, skin changes such as dryness and hyperkeratosis, and decreased or absent pulses in the affected arteries. Buerger’s disease and obliterating endarteritis are other names for thromboangiitis obliterans, which is a rare inflammatory disease that affects the small and medium-sized arteries of the extremities, typically in young smokers. The symptoms of thromboangiitis obliterans may include pain, tenderness, and ulcers in the affected limb, as well as absence of pulses in the affected arteries. Thrombosis of the femoral artery is a blockage of the femoral artery, which is a major artery that supplies blood to the thigh and lower leg. Symptoms of femoral artery thrombosis may include sudden onset of severe pain, swelling, and coolness in the affected limb, as well as absence of pulses in the affected artery. Raynaud’s disease is a condition that causes episodes of reduced blood flow to the fingers and toes, typically triggered by cold temperatures or stress. Symptoms of Raynaud’s disease may include numbness, tingling, and color changes in the affected digits. Therefore, based on the information provided in the question, the most likely diagnosis for the 58-year-old patient who complains of pain in the left lower extremity, which worsens when walking, a feeling of cooling and numbness in both feet, with pale, dry skin, hyperkeratosis, poorly developed hair cover and positive “furrow” symptom on the left lower leg, and absence of pulse on the arteries of the foot and popliteal artery and weakened pulse on the femoral artery, is obliterating atherosclerosis of the arteries of the lower extremities. |
33. During the planned AKDP vaccination, the child suddenly developed severe restlessness, signs of pain reaction, shortness of breath, wheezing, marbling of the skin, and cold sweat. Objectively: consciousness is dimmed, heart rate – 150/min, blood pressure – 60/40 mm Hg, deafness of heart sounds. The child was diagnosed with anaphylactic shock. Which drug should be administered first? Select one:
A. Analgin
B. Euphilinus
C. Lasix (Furosemide)
D. Adrenalin
E. Suprastin
The correct answer is: Adrenaline
Explanation
The first drug that should be administered in the case of a child who suddenly develops severe restlessness, signs of pain reaction, shortness of breath, wheezing, marbling of the skin, cold sweat, dimmed consciousness, tachycardia, hypotension, and deafness of heart sounds during the planned AKDP vaccination, and is diagnosed with anaphylactic shock, is adrenaline (epinephrine). Anaphylactic shock is a severe and potentially life-threatening allergic reaction that can occur in response to an allergen, such as a vaccine. Symptoms of anaphylactic shock may include restlessness, pain, shortness of breath, wheezing, skin changes, cold sweat, altered consciousness, tachycardia, and hypotension. Anaphylactic shock requires immediate medical attention and treatment with adrenaline, which is a potent vasoconstrictor and bronchodilator that can rapidly reverse the symptoms of anaphylaxis. Analgin, Euphilinus, Lasix (Furosemide), and Suprastin are not appropriate first-line treatments for anaphylactic shock. Analgin is a pain reliever, Euphilinus is a bronchodilator, Lasix (Furosemide) is a diuretic, and Suprastin is an antihistamine. While these drugs may have a role in the management of anaphylactic shock, they should not be used as the first-line treatment. Adrenaline is the most effective and important initial treatment for anaphylactic shock. Therefore, based on the information provided in the question, the first drug that should be administered in the case of a child who suddenly develops severe restlessness, signs of pain reaction, shortness of breath, wheezing, marbling of the skin, cold sweat, dimmed consciousness, tachycardia, hypotension, and deafness of heart sounds during the planned AKDP vaccination, and is diagnosed with anaphylactic shock, is adrenaline (epinephrine). Immediate medical attention is needed to administer adrenaline and provide supportive care. |
34. At night, a 63-year-old woman had a sudden attack of dysentery. She has been suffering from hypertension for about 15 years, and suffered a myocardial infarction 2 years ago. Objectively: the position in bed is orthopnea, the skin is pale, the patient is covered in cold sweat, acrocyanosis. Pulse – 104/min. Blood pressure – 210/130 mm Hg, respiratory rate – 38/min. The percussion sound is pulmonary, dulled in the lower parts, isolated dry rales are heard throughout, in the lower parts – voiceless, small-vesicular. What complication most likely developed in the patient? Select one:
A. Acute left ventricular failure
B. Thromboembolism of the pulmonary artery
C. Acute right ventricular failure
D. An attack of bronchial asthma
E. Paroxysmal tachycardia
The correct answer is: Acute left ventricular failure
Explanation
The most likely complication that developed in the 63-year-old woman who had a sudden attack of dyspnea at night, with a history of hypertension and myocardial infarction, and presenting with orthopnea, pale skin, cold sweat, acrocyanosis, tachycardia, elevated blood pressure, respiratory distress, and pulmonary findings such as dullness in the lower parts, isolated dry rales throughout, and voiceless, small-vesicular rales in the lower parts, is acute left ventricular failure. Acute left ventricular failure is a condition in which the heart is unable to pump enough blood to meet the body’s needs, leading to a buildup of fluid in the lungs and pulmonary congestion. Symptoms of acute left ventricular failure may include orthopnea, dyspnea, tachycardia, elevated blood pressure, and signs of respiratory distress, such as pulmonary findings on physical examination. Thromboembolism of the pulmonary artery is a blockage of the pulmonary artery by a blood clot, which can cause sudden onset of dyspnea, chest pain, and hemoptysis. However, the presence of pulmonary findings on physical examination and elevated blood pressure make acute left ventricular failure a more likely diagnosis in this case. Acute right ventricular failure is a condition in which the heart is unable to pump blood effectively to the lungs, leading to a buildup of fluid in the body and peripheral edema. While acute right ventricular failure can sometimes occur as a complication of acute left ventricular failure, the clinical presentation in this case is more suggestive of left-sided heart failure. An attack of bronchial asthma can cause dyspnea, wheezing, and chest tightness, but it is unlikely to cause the other symptoms and physical findings observed in this case. Paroxysmal tachycardia is a condition in which the heart beats abnormally fast, but it is not typically associated with the other symptoms and physical findings observed in this case. Therefore, based on the information provided in the question, the most likely complication that developed in the 63-year-old woman who had a sudden attack of dyspnea at night, with a history of hypertension and myocardial infarction, and presenting with orthopnea, pale skin, cold sweat, acrocyanosis, tachycardia, elevated blood pressure, respiratory distress, and pulmonary findings such as dullness in the lower parts, isolated dry rales throughout, and voiceless, small-vesicular rales in the lower parts, is acute left ventricular failure. Urgent medical attention is needed to confirm the diagnosis and initiate appropriate treatment, which may include oxygen therapy, diuretics, and medications to improve cardiac function. |
35. Several chemical substances enter the human body from atmospheric air. What is the type of combined effect called, where the combined effect exceeds the sum of the effects of each individual substance included in the combination, in the case of its isolated effect on the body?
a. Potentiation
b. Complex action
c. Antagonism
d. Isolated action
e. Combined action
The correct answer is: Potentiation
Explanation
The most likely complication that developed in the 63-year-old woman who had a sudden attack of dyspnea at night, with a history of hypertension and myocardial infarction, and presenting with orthopnea, pale skin, cold sweat, acrocyanosis, tachycardia, elevated blood pressure, respiratory distress, and pulmonary findings such as dullness in the lower parts, isolated dry rales throughout, and voiceless, small-vesicular rales in the lower parts, is acute left ventricular failure. Acute left ventricular failure is a condition in which the heart is unable to pump enough blood to meet the body’s needs, leading to a buildup of fluid in the lungs and pulmonary congestion. Symptoms of acute left ventricular failure may include orthopnea, dyspnea, tachycardia, elevated blood pressure, and signs of respiratory distress, such as pulmonary findings on physical examination. Thromboembolism of the pulmonary artery is a blockage of the pulmonary artery by a blood clot, which can cause sudden onset of dyspnea, chest pain, and hemoptysis. However, the presence of pulmonary findings on physical examination and elevated blood pressure make acute left ventricular failure a more likely diagnosis in this case. Acute right ventricular failure is a condition in which the heart is unable to pump blood effectively to the lungs, leading to a buildup of fluid in the body and peripheral edema. While acute right ventricular failure can sometimes occur as a complication of acute left ventricular failure, the clinical presentation in this case is more suggestive of left-sided heart failure. An attack of bronchial asthma can cause dyspnea, wheezing, and chest tightness, but it is unlikely to cause the other symptoms and physical findings observed in this case. Paroxysmal tachycardia is a condition in which the heart beats abnormally fast, but it is not typically associated with the other symptoms and physical findings observed in this case. Therefore, based on the information provided in the question, the most likely complication that developed in the 63-year-old woman who had a sudden attack of dyspnea at night, with a history of hypertension and myocardial infarction, and presenting with orthopnea, pale skin, cold sweat, acrocyanosis, tachycardia, elevated blood pressure, respiratory distress, and pulmonary findings such as dullness in the lower parts, isolated dry rales throughout, and voiceless, small-vesicular rales in the lower parts, is acute left ventricular failure. Urgent medical attention is needed to confirm the diagnosis and initiate appropriate treatment, which may include oxygen therapy, diuretics, and medications to improve cardiac function. |
36. A 12-year-old child has an attack of frequent heartbeat. During examination: pallor, pulsation of jugular veins, heart rate – 210/min. On the ECG – supraventricular tachycardia. What treatment method should be used in the first place? Select one:
A. Mechanical influence on n.vagus
B. Intravenous administration of ATP
C. Electropulse therapy
D. Intravenous administration of isoptin
E. Intravenous administration of novocaine
The correct answer is: Mechanical influence on n.vagus
Explanation
The first-line treatment method that should be used in the case of a 12-year-old child with an attack of frequent heartbeat, pallor, pulsation of jugular veins, heart rate of 210/min, and a diagnosis of supraventricular tachycardia on the ECG, is mechanical influence on the vagus nerve. Supraventricular tachycardia (SVT) is an abnormally fast heart rhythm that originates above the ventricles. Symptoms of SVT may include palpitations, chest discomfort, shortness of breath, and dizziness. The first-line treatment for SVT is often the use of vagal maneuvers, which are non-invasive techniques that stimulate the vagus nerve and can slow down the heart rate. Mechanical influence on the vagus nerve is a type of vagal maneuver that involves applying pressure to the eyes, the carotid sinus, or the abdomen to stimulate the vagus nerve and slow down the heart rate. Examples of mechanical vagal maneuvers include the Valsalva maneuver, the carotid sinus massage, and the diving reflex. Intravenous administration of ATP, isoptin, or novocaine, or electropulse therapy may be used in the treatment of SVT if vagal maneuvers are ineffective or contraindicated. ATP is a medication that can slow down the heart rate by blocking the conduction of electrical impulses through the atrioventricular (AV) node. Isoptin (verapamil) is a calcium channel blocker that can also block the conduction of electrical impulses through the AV node. Novocaine (lidocaine) is a local anesthetic that can be used to suppress ectopic beats. Electropulse therapy involves the use of electrical shocks to reset the heart’s rhythm. Therefore, based on the information provided in the question, the first-line treatment method that should be used in the case of a 12-year-old child with an attack of frequent heartbeat, pallor, pulsation of jugular veins, heart rate of 210/min, and a diagnosis of supraventricular tachycardia on the ECG, is mechanical influence on the vagus nerve, such as the Valsalva maneuver or carotid sinus massage. Urgent medical attention is needed to confirm the diagnosis and initiate appropriate treatment. |
37. A 45-year-old woman complains of an attack-like unbearable pain in the left half of the face lasting 1-2 minutes. Attacks are provoked by chewing. She fell ill two months ago after hypothermia. Objectively: pain at the exit points of the trigeminal nerve on the left. A touch near the wing of the nose on the left causes another attack with a tonic spasm of the facial muscles. Which of the listed diagnoses is the most likely? Select one:
A. Sinusitis
B. Neuralgia of the glossopharyngeal nerve
C. Arthritis of the mandibular joint
D. Facial migraine
E. Trigeminal neuralgia
The correct answer is: Neuralgia of the trigeminal nerve
Explanation
The most likely diagnosis in the case of a 45-year-old woman who complains of an attack-like unbearable pain in the left half of the face lasting 1-2 minutes, which is provoked by chewing, and has pain at the exit points of the trigeminal nerve on the left, with a touch near the wing of the nose on the left causing another attack with a tonic spasm of the facial muscles, is trigeminal neuralgia. Trigeminal neuralgia is a condition that affects the trigeminal nerve, which is responsible for sensation in the face. It is characterized by sudden, severe, and recurring attacks of pain in the face, typically on one side only. The pain is often triggered by activities such as chewing, speaking, or touching the face, and can be described as sharp, stabbing, or electric shock-like. The pain usually lasts for a few seconds to a few minutes and can be debilitating. Sinusitis is an inflammation of the sinuses, which can cause facial pain, pressure, and congestion, but it is unlikely to cause the sudden, severe, and recurring attacks of pain described in this case. Neuralgia of the glossopharyngeal nerve is a rare condition that affects the glossopharyngeal nerve, which is responsible for sensation in the throat and tongue. It can cause severe pain in the throat, ear, and tongue, but it is unlikely to cause the pain in the face described in this case. Arthritis of the mandibular joint is a condition that affects the jaw joint, which can cause pain, stiffness, and difficulty chewing, but it is unlikely to cause the sudden, severe, and recurring attacks of pain described in this case. Facial migraine is a type of migraine headache that affects the face, but it typically causes a more diffuse and longer-lasting pain than the sudden and severe attacks of pain described in this case. Therefore, based on the information provided in the question, the most likely diagnosis in the case of a 45-year-old woman who complains of an attack-like unbearable pain in the left half of the face lasting 1-2 minutes, which is provoked by chewing, and has pain at the exit points of the trigeminal nerve on the left, with a touch near the wing of the nose on the left causing another attack with a tonic spasm of the facial muscles, is trigeminal neuralgia. Urgent medical attention is needed to confirm the diagnosis and initiate appropriate treatment, which may include medications to control pain, such as carbamazepine or gabapentin. |
38. A 38-year-old man works in the zone of action of ionizing radiation. During periodic medical examination, he does not present any complaints. In the blood: iron – $4.5cdot 10^{12/$l, Hb – 80 g/l, leuk. – $2.8cdot 10^9$/l, platelets – $30cdot 10^9$ / l. Can this person be allowed to work with sources of ionizing radiation? Select one:
A. Work with radioactive substances and other sources of ionizing radiation is contraindicated
B. Can work only with low-level radioactive substances
C. It is allowed to work with radioactive substances with a limitation of working time
D. Allowed to work after an extended medical examination
E. It is allowed to work with radioactive substances
The correct answer is: Work with radioactive substances and other sources of ionizing radiation is contraindicated
Explanation
Based on the information provided in the question, the 38-year-old man who works in the zone of action of ionizing radiation has low levels of red blood cells (RBCs), hemoglobin (Hb), leukocytes, and platelets in his blood. This suggests that he may have developed radiation-induced bone marrow suppression, which can occur as a result of exposure to ionizing radiation. Bone marrow suppression can lead to decreased production of blood cells, which can result in anemia, leukopenia, and thrombocytopenia. These conditions can increase the risk of bleeding, infection, and other complications, particularly if the individual is exposed to additional sources of radiation. Therefore, in this case, work with radioactive substances and other sources of ionizing radiation is contraindicated. The individual should be advised to avoid exposure to ionizing radiation and should undergo further medical evaluation and treatment to manage the radiation-induced bone marrow suppression. The individual may also need to be monitored regularly for signs of radiation-induced health effects and should receive appropriate medical care if any symptoms or complications arise. |
39. A 57-year-old woman notes weakness, shortness of breath, lack of appetite, loose stools. He has been sick for two years. Objectively: the skin is pale, the sclera is subicteric, the tongue is bright red, with cracks. Lymph nodes are not enlarged. Pulse – 100/min. Blood pressure – 105/70 mm Hg. The liver is +3 cm, the spleen is not palpable. In the blood: erythrocytes – 1.2 10 /l, Hb – 56 g/l, CP – 1.4, macrocytes, leukocytes – 2.5 10 /l, Eo1%, yu- 1%, metamyelocytes – 1%, p- 8%, c- 47%, lymph.- 38%, mon.- 4%, reticul.- 0.1%, platelets – 100 10 /l, erythrocyte sedimentation rate – 20 mm/h, indirect bilirubin – 26 mmol/l. What changes in bone marrow punctate should be expected? Select one:
A. Predominance of lymphoid tissue
B. Predominance of megaloblasts
C. Hyperplasia of the erythroid sprout
D. An increase in the number of sideroblasts
E. The presence of blast cells . 12 . 9 . 9
The correct answer is: Predominance of megaloblasts
Explanation
The clinical presentation in the case of a 57-year-old woman with weakness, shortness of breath, lack of appetite, loose stools, pale skin, subicteric sclera, bright red tongue with cracks, and pancytopenia (low red blood cells, white blood cells, and platelets) suggests the possibility of megaloblastic anemia, which is a type of anemia caused by impaired DNA synthesis due to a deficiency of vitamin B12 or folic acid. The presence of macrocytes and reticulocytopenia in the blood smear also supports the diagnosis of megaloblastic anemia. Therefore, changes in the bone marrow punctate in this case are expected to show a predominance of megaloblasts. Megaloblasts are large, immature red blood cell precursors that are characteristic of megaloblastic anemia. The megaloblastic changes in the bone marrow reflect the impaired DNA synthesis and abnormal maturation of red blood cells that occur in this condition. Predominance of lymphoid tissue in the bone marrow punctate is not expected in this case, as there is no evidence of lymph node enlargement or lymphocytosis in the blood. Hyperplasia of the erythroid sprout is a feature of some types of anemia, but it is not specific to megaloblastic anemia. An increase in the number of sideroblasts may be seen in some types of anemia, but it is not specific to megaloblastic anemia. The presence of blast cells in the bone marrow punctate is not expected in this case, as there is no evidence of acute leukemia in the blood or clinical presentation. In summary, based on the information provided in the question, the changes in bone marrow punctate that should be expected in the case of a 57-year-old woman with weakness, shortness of breath, lack of appetite, loose stools, pale skin, subicteric sclera, bright red tongue with cracks, and pancytopenia are a predominance of megaloblasts, which is characteristic of megaloblastic anemia. |
40. A 32-year-old patient complains of pain in the lower limbs, especially when walking, intermittent lameness, a feeling of numbness in the toes, coldness of the limbs, inability to walk more than 100 m. He sleeps with his leg down. The patient has been smoking since the age of 16, abuses alcohol. The left lower extremity is colder than the right, the skin of the extremities is dry, the pulsation on the arteries of the feet is not determined, on the femoral arteries it is preserved. What is the most likely diagnosis? Select one:
A. Lerich syndrome
B. Diabetic angiopathy
C. Deep thrombophlebitis
D. Raynaud’s disease
E. Obliterating endarteritis
The correct answer is: Obliterating endarteritis
Explanation
The most likely diagnosis in the case of a 32-year-old patient who complains of pain in the lower limbs, especially when walking, intermittent lameness, a feeling of numbness in the toes, coldness of the limbs, inability to walk more than 100 m, and has a history of smoking since the age of 16 and alcohol abuse, is obliterating endarteritis, also known as peripheral arterial disease (PAD). Obliterating endarteritis is a chronic arterial disease that affects the arteries of the extremities, leading to reduced blood flow and ischemia. It is commonly associated with risk factors such as smoking, diabetes, hypertension, and hyperlipidemia. The symptoms of obliterating endarteritis typically include pain in the lower limbs, especially during physical activity, intermittent claudication (lameness), numbness, and coldness of the limbs. In severe cases, the affected limb may develop ulcers or gangrene. The absence of pulsation on the arteries of the feet in this case suggests that the patient may have significant arterial occlusion, which can lead to tissue ischemia and damage. The preservation of pulsation on the femoral arteries suggests that the occlusion is likely located distal to the femoral artery. Lerich syndrome is a combination of symptoms that includes bilateral intermittent claudication, impotence, and decreased or absent pulses in the lower limbs. This syndrome is often associated with aortoiliac occlusive disease and is less likely to occur in a 32-year-old patient. Diabetic angiopathy can cause peripheral arterial disease, but it is less likely to occur in a patient of this age who does not have a history of diabetes. Deep thrombophlebitis can cause pain and swelling in the lower limbs, but it is less likely to cause the symptoms of intermittent claudication, numbness, and coldness described in this case. Raynaud’s disease is a condition that causes spasm of the small arteries in the fingers and toes, leading to pain, numbness, and color changes in the affected areas. It is less likely to cause the symptoms of intermittent claudication and reduced pulses described in this case. Therefore, based on the information provided in the question, the most likely diagnosis in the case of a 32-year-old patient who complains of pain in the lower limbs, especially when walking, intermittent lameness, a feeling of numbness in the toes, coldness of the limbs, inability to walk more than 100 m, and has a history of smoking since the age of 16 and alcohol abuse, is obliterating endarteritis. Urgent medical attention is needed to confirm the diagnosis and initiate appropriate treatment, which may include lifestyle modifications, medications to improve blood flow, and, in severe cases, revascularization procedures. |
41. A 35-year-old patient complains about the presence of enlarged peripheral lymph nodes for a long time, which do not bother him. From the anamnesis of the disease: initially the lymph nodes of the neck, supraclavicular, axillary region increased, new groups of lymph nodes appeared. Objectively: lymph nodes during palpation are soft-elastic, enlarged, painless, not fused with the surrounding tissues. Which research method is the most informative for early diagnosis of the disease? Select one:
A. Puncture biopsy
B. Radioisotope scanning of the skeleton
C. Ultrasound examination
D. X-ray examination
E. Magnetic resonance imaging
The correct answer is: Puncture biopsy
Explanation
In the case of a 35-year-old patient who complains of the presence of enlarged peripheral lymph nodes for a long time, which do not bother him, and has soft-elastic, painless, and not fused with the surrounding tissues lymph nodes on palpation, the most informative research method for early diagnosis of the disease is a puncture biopsy. A puncture biopsy involves inserting a needle into the enlarged lymph node and removing a small sample of tissue for examination under a microscope. This procedure can help to determine the nature of the lymph node enlargement, whether it is due to inflammation or cancer. While other imaging modalities such as ultrasound, X-ray, and magnetic resonance imaging (MRI) can provide valuable information about the size, location, and characteristics of the enlarged lymph nodes, they may not be able to distinguish between benign and malignant conditions. Radioisotope scanning of the skeleton is a nuclear medicine imaging technique that can help to detect areas of abnormal bone metabolism, but it is not specific to lymph node enlargement. Therefore, based on the information provided in the question, the most informative research method for early diagnosis of the disease in the case of a 35-year-old patient who complains of the presence of enlarged peripheral lymph nodes for a long time is a puncture biopsy. This procedure can help to determine whether the lymph node enlargement is due to a benign or malignant condition, and can guide further diagnostic and treatment decisions. |
42. The 56-year-old patient, an engineer, was infected with syphilis at the age of 35 and was treated with “folk methods”. About 5 years ago, he became forgetful, did not cope with work, cynically joked, bought junk, collected cigarette butts on the street. Objectively: indifferent, speech is slowed down, dysarthric, judgments are primitive, cannot perform simple arithmetic operations, explain simple metaphors. Sloppy, not interested in anything, idle. He considers himself completely healthy. Describe the mental state of the patient: Select one:
A. Stunnedness
B. Total dementia
C. Korsak (amnestic) syndrome
D. Hysterical pseudodementia
E. Lacunar (dysmnestic) dementia
The correct answer is: Total dementia
Explanation
The mental state of the 56-year-old patient described in the question is consistent with total dementia. Total dementia is a severe and progressive decline in cognitive function that affects multiple areas of mental ability, including memory, language, judgment, and reasoning. The symptoms of total dementia include forgetfulness, difficulty with problem-solving, impaired judgment and decision-making, disorientation to time and place, and changes in personality and behavior. In the case of the patient described in the question, the symptoms of forgetfulness, slowed speech, dysarthria (difficulty with articulation), primitive judgments, inability to perform simple arithmetic operations, and decreased interest in activities are all consistent with total dementia. The patient’s lack of insight into his condition and belief that he is completely healthy is also typical of dementia. Korsakoff’s syndrome is a type of amnestic syndrome that is commonly associated with chronic alcoholism and thiamine deficiency. It is characterized by anterograde and retrograde amnesia, confabulation (the production of fabricated, distorted, or misinterpreted memories), and other cognitive impairments. The patient in the question does not exhibit the characteristic symptoms of Korsakoff’s syndrome. Hysterical pseudodementia is a condition in which a patient presents with symptoms of dementia but does not have a true cognitive decline. It is typically associated with underlying psychological or emotional disturbances and is not consistent with the progressive cognitive decline observed in the patient in the question. Lacunar dementia is a type of vascular dementia that is characterized by small infarcts or lesions in the deep structures of the brain, resulting in cognitive and motor impairments. The symptoms of lacunar dementia are not consistent with the progressive decline in cognitive function observed in the patient in the question. Therefore, based on the information provided in the question, the most likely diagnosis for the mental state of the 56-year-old patient is total dementia, which is characterized by a severe and progressive decline in cognitive function affecting multiple areas of mental ability. |
43. A 65-year-old patient complains of shortness of breath, a strong cough with a small amount of sputum with streaks of blood, weight loss, body temperature – 37.2°C, loss of appetite, weakness. He has been ill for many years, his condition worsened a year ago, and shortness of breath appeared 3 weeks ago. He smokes all his life, works as a carpenter. Objectively: normal build, exhausted. Sinking of the right half of the chest, limitation of excursion, participation of additional muscles in breathing, respiratory rate – 22/min. Percussively above the right upper part of the dulling of the sound. On X-ray OGK: the upper right lobe is reduced in size, homogeneous darkening above it, associated with the root, the root is deformed, mediastinal organs are slightly shifted to the right. What is the most likely diagnosis? Select one:
A. Right-sided total pneumothorax
B. Fibrosing alveolitis
C. Obturational atelectasis of the lungs
D. Pulmonary sarcoidosis
E. Pulmonary tuberculosis
The correct answer is: Obstructive atelectasis of the lungs
Explanation
The most likely diagnosis for the 65-year-old patient described in the question, who complains of shortness of breath, a strong cough with a small amount of sputum with streaks of blood, weight loss, loss of appetite, weakness, and has a history of smoking and working as a carpenter, is obstructive atelectasis of the lungs. Atelectasis is a condition in which a part or the whole of the lung collapses, resulting in a reduction or loss of lung function. Obstructive atelectasis occurs when there is an obstruction of the airways, preventing air from reaching the affected part of the lung. The symptoms of obstructive atelectasis typically include shortness of breath, cough, chest pain, and fever, and can be accompanied by sputum production and weight loss. The chest examination may reveal a decrease in chest movement on the affected side, a dull percussion sound, and decreased breath sounds. The X-ray findings of a reduced size of the upper right lobe, homogeneous darkening above it, associated with the root, deformed root, and mediastinal organs slightly shifted to the right are consistent with obstructive atelectasis. While other conditions such as fibrosing alveolitis, pulmonary sarcoidosis, and pulmonary tuberculosis can also present with respiratory symptoms and abnormal X-ray findings, the clinical presentation and X-ray findings described in the question are most consistent with obstructive atelectasis. Right-sided total pneumothorax is unlikely in this case, as it would typically present with acute onset of shortness of breath, chest pain, and decreased breath sounds on the affected side, and would be accompanied by a hyper-resonant percussion sound. Therefore, based on the information provided in the question, the most likely diagnosis for the 65-year-old patient who complains of shortness of breath, a strong cough with a small amount of sputum with streaks of blood, weight loss, loss of appetite, weakness, and has a history of smoking and working as a carpenter, is obstructive atelectasis of the lungs. Urgent medical attention is needed to confirm the diagnosis and initiate appropriate treatment, which may include bronchoscopy, chest physiotherapy, and, in severe cases, surgery. |
44. An 8-year-old girl complains of frequent, painful urination, in small portions, urinary incontinence. He has been sick for the 2nd day, the disease is associated with hypothermia. Pasternacki’s symptom is negative. General blood analysis without pathological changes, in the general analysis of urine: leukocytes – 20-30 in the field of vision, erythrocytes – 40-50 in the field of vision, unchanged, bacteriuria. Specify the most likely diagnosis: Select one:
A. Urinary stone disease
B. Vulvitis
C. Glomerulonephritis
D. Cystitis
E. Pyelonephritis
The correct answer is: Cystitis
Explanation
The most likely diagnosis for the 8-year-old girl described in the question, who complains of frequent, painful urination, urinary incontinence, and has leukocytes, erythrocytes, and bacteriuria in the urine analysis, is cystitis. Cystitis is an inflammation of the bladder that is commonly caused by a bacterial infection. The symptoms of cystitis typically include painful and frequent urination, a feeling of urgency to urinate, and lower abdominal pain or discomfort. In some cases, there may also be blood in the urine. The negative Pasternacki’s symptom (absence of pain on percussion of the lower back) suggests that the kidney is not involved, making pyelonephritis less likely. Urinary stone disease and vulvitis are also unlikely in this case, as they do not typically present with the characteristic symptoms of painful and frequent urination. Glomerulonephritis is a condition in which there is inflammation of the glomeruli (small blood vessels) in the kidneys, resulting in impaired kidney function. While glomerulonephritis can present with urinary abnormalities such as blood and protein in the urine, it is less likely to cause the characteristic symptoms of painful and frequent urination. Therefore, based on the information provided in the question, the most likely diagnosis for the 8-year-old girl who complains of frequent, painful urination, in small portions, urinary incontinence, and has leukocytes, erythrocytes, and bacteriuria in the urine analysis, is cystitis. Urgent medical attention is needed to confirm the diagnosis and initiate appropriate treatment, which typically includes antibiotics and pain relief medication, as well as measures to prevent recurrence of the infection. |
45. A 22-year-old patient complained about the absence of menstruation for 8 months. From the anamnesis: menarche since the age of 12. Since the age of 18, menstruation has been irregular. There were no pregnancies. The mammary glands are developed correctly, the release of drops of milk from the nipples when pressed. During gynecological examination: moderate uterine hypoplasia. During hormonal research: the level of prolactin is 2 times higher than the norm. At the computer tomography in the area of the Turkish saddle – a volume formation with a diameter of 4 mm. What is the most likely diagnosis? Select one:
A. Sheehan syndrome
B. Stein-Leventhal syndrome
C. Pituitary tumor
D. Lactational amenorrhea
E. Itsenko-Cushing’s disease
The correct answer is: Pituitary tumor
Explanation
The most likely diagnosis for the 22-year-old patient who complains of absence of menstruation for 8 months, has moderate uterine hypoplasia, elevated prolactin levels, and a volume formation with a diameter of 4 mm in the area of the Turkish saddle on CT scan is pituitary tumor. A pituitary tumor can cause a range of symptoms depending on the size and location of the tumor. If the tumor is located near the pituitary stalk, it can cause elevated levels of prolactin, leading to a condition called hyperprolactinemia. Hyperprolactinemia can interfere with the normal production of estrogen and progesterone, leading to menstrual irregularities and, in some cases, amenorrhea. The presence of moderate uterine hypoplasia suggests that the patient’s reproductive system has not fully developed, which could be due to the effects of elevated prolactin levels. The release of drops of milk from the nipples when pressed is also a sign of hyperprolactinemia. The CT scan findings of a volume formation with a diameter of 4 mm in the area of the Turkish saddle is consistent with a pituitary tumor, which can cause hyperprolactinemia and menstrual irregularities. Sheehan syndrome is a rare condition that can occur as a result of severe postpartum hemorrhage, leading to pituitary gland necrosis and hormonal deficiencies. Stein-Leventhal syndrome (also known as polycystic ovary syndrome) is a condition that can cause menstrual irregularities and infertility, but it is not typically associated with elevated prolactin levels. Lactational amenorrhea is a temporary cessation of menstruation that occurs during breastfeeding. Itsenko-Cushing’s disease is a rare condition that results in excess cortisol production, leading to a range of symptoms including menstrual irregularities, but it is not typically associated with hyperprolactinemia or uterine hypoplasia. Therefore, based on the information provided in the question, the most likely diagnosis for the 22-year-old patient who complains of absence of menstruation for 8 months, has moderate uterine hypoplasia, elevated prolactin levels, and a volume formation with a diameter of 4 mm in the area of the Turkish saddle on CT scan is pituitary tumor. Urgent medical attention is needed to confirm the diagnosis and initiate appropriate treatment, which may include medications to reduce prolactin levels and/or surgery to remove the tumor. |
46. A 33-year-old patient complained of shortness of breath during physical exertion, palpitations, heart failure, and leg swelling. As a child, he suffered from acute rheumatic fever, for which he was hospitalized. In the future, he did not seek medical help. Objectively: Ps- 92/min., rhythmic. Blood pressure – 110/70 mm Hg. Above the apex – I tone is increased, three-part rhythm, diastolic noise. What heart defect should you think about first? Select one:
A. Stenosis of the mouth of the aorta
B. Stenosis of the mitral valve
C. Insufficiency of the aortic valve
D. Stenosis of the tricuspid valve
E. Insufficiency of the mitral valve
The correct answer is: Stenosis of the mitral valve
Explanation
The most likely heart defect for the 33-year-old patient who complains of shortness of breath during physical exertion, palpitations, heart failure, and leg swelling, and who has an increased first heart sound, a three-part rhythm, and diastolic noise above the apex, is stenosis of the mitral valve. Acute rheumatic fever (ARF) is a systemic inflammatory disease that can develop as a complication of untreated streptococcal pharyngitis. ARF can cause damage to the heart valves, including the mitral valve, which can lead to stenosis or insufficiency. Mitral stenosis is the most common valvular lesion caused by ARF. The symptoms of mitral stenosis typically include shortness of breath, palpitations, fatigue, and leg swelling, which are often worsened by physical exertion. The increased first heart sound, three-part rhythm, and diastolic noise above the apex are typical physical examination findings in patients with mitral stenosis. Stenosis of the mouth of the aorta, insufficiency of the aortic valve, stenosis of the tricuspid valve, and insufficiency of the mitral valve are less likely in this case, as they do not typically present with the characteristic symptoms and physical examination findings of mitral stenosis. Therefore, based on the information provided in the question, the most likely heart defect for the 33-year-old patient who complains of shortness of breath during physical exertion, palpitations, heart failure, and leg swelling, and who has an increased first heart sound, a three-part rhythm, and diastolic noise above the apex, is stenosis of the mitral valve. Urgent medical attention is needed to confirm the diagnosis and initiate appropriate treatment, which may include medications to control symptoms and prevent complications, as well as surgical intervention to repair or replace the damaged valve. |
47. A 23-year-old patient with type I diabetes developed nausea and vomiting in the second week of community-acquired pneumonia. She lost consciousness in the evening. Hospitalized. Objectively: the skin is dry and pale. Breathing is noisy, the tongue is dry, with a layer of brown color. Heart rate – 129/min., blood pressure – 85/50 mm Hg. Does not respond to abdominal palpation. Liver +3 cm. The reaction to acetone is sharply positive, blood glucose – 26 mmol/l. Previous diagnosis: Select one:
A. Hyperosmolar coma
B. Lactacidemic coma
C. Hepatic coma
D. Infectious-toxic shock
E. Ketoacidotic coma
The correct answer is: Ketoacidotic coma
Explanation
The most likely diagnosis for the 23-year-old patient with type I diabetes who developed nausea and vomiting in the second week of community-acquired pneumonia, lost consciousness, and was found to have dry and pale skin, noisy breathing, dry and brown tongue, tachycardia, hypotension, hepatomegaly, and a sharply positive reaction to acetone with a blood glucose level of 26 mmol/L is ketoacidotic coma. Ketoacidotic coma is a serious complication of uncontrolled diabetes, which can occur when there is a shortage of insulin in the body, leading to high levels of glucose in the blood and the breakdown of fatty acids into ketone bodies. The accumulation of ketone bodies can lead to a decrease in blood pH, resulting in metabolic acidosis and a wide range of symptoms such as nausea, vomiting, dehydration, altered consciousness, and hypotension. In the case of the patient described in the question, the presence of type I diabetes, the sharply positive reaction to acetone, and the high blood glucose level are all consistent with ketoacidotic coma. The hepatomegaly may be due to fatty liver, which can occur as a result of uncontrolled diabetes. The respiratory distress and brown tongue may be signs of severe dehydration and acidosis. Hyperosmolar coma, lactacidemic coma, and hepatic coma are less likely in this case, as they do not typically present with the characteristic symptoms of high blood glucose, positive acetone reaction, and metabolic acidosis that are observed in ketoacidotic coma. Infectious-toxic shock is also less likely in this case, as the symptoms described are more consistent with the complications of uncontrolled diabetes rather than an acute infection. Therefore, based on the information provided in the question, the most likely diagnosis for the 23-year-old patient with type I diabetes who developed nausea and vomiting in the second week of community-acquired pneumonia, lost consciousness, and was found to have dry and pale skin, noisy breathing, dry and brown tongue, tachycardia, hypotension, hepatomegaly, and a sharply positive reaction to acetone with a blood glucose level of 26 mmol/L is ketoacidotic coma. Urgent medical attention is needed to confirm the diagnosis and initiate appropriate treatment, which typically includes insulin therapy, fluid and electrolyte replacement, and treatment of any underlying infection or other contributing factors. |
48. A 30-year-old patient came to see a dermatologist with complaints of a rash and severe itching, especially at night. Rash appeared 2 weeks ago after traveling. Objectively: on the skin of the lateral surfaces of the fingers, hands, in the area of the wrists, elbow joints, the lower part of the abdomen, genitals, and buttocks, papulovesicular elements, single pustules, and ulcers are present in pairs. What disease can be assumed? Select one:
A. Eczema
B. Shingles
C. Pyoderma
D. Scabies
E. Dermatitis
The correct answer is: Corosta
Explanation
The most likely diagnosis for the 30-year-old patient who complains of a rash and severe itching, especially at night, and who has papulovesicular elements, single pustules, and ulcers in pairs on the lateral surfaces of the fingers, hands, wrists, elbow joints, lower part of the abdomen, genitals, and buttocks, after traveling, is scabies. Scabies is a contagious skin condition caused by the Sarcoptes scabiei mite. The mites burrow into the skin to lay eggs, causing a rash and severe itching, especially at night. The papulovesicular elements, pustules, and ulcers in pairs are characteristic of scabies, as are the areas of the body affected, which typically include the fingers, wrists, elbows, genitals, and buttocks. Eczema, shingles, pyoderma, and dermatitis are less likely in this case, as they do not typically present with the characteristic symptoms and physical examination findings of scabies. Eczema is a chronic skin condition characterized by dry, itchy, and inflamed skin. Shingles is a viral infection caused by the varicella-zoster virus, which typically causes a painful rash on one side of the body. Pyoderma is a bacterial skin infection that can cause pustules and ulcers, but it typically presents with different distribution and morphology of lesions. Dermatitis is a general term for inflammation of the skin, which can have a wide range of causes and symptoms. Therefore, based on the information provided in the question, the most likely diagnosis for the 30-year-old patient who complains of a rash and severe itching, especially at night, and who has papulovesicular elements, single pustules, and ulcers in pairs on the lateral surfaces of the fingers, hands, wrists, elbow joints, lower part of the abdomen, genitals, and buttocks, after traveling, is scabies. Urgent medical attention is needed to confirm the diagnosis and initiate appropriate treatment, which may include topical or oral medications to kill the mites and relieve symptoms, as well as measures to prevent the spread of the infection to others. |
49. A 41-year-old patient, a hunter, complains of heaviness in the area of the right hypochondrium. There are no other complaints. In the anamnesis: a year ago, causeless urticaria and itching of the skin. Objectively: the edge of the liver is palpable 3 cm below the costal arch, rounded, painless. No other pathology could be detected during the physical examination of the patient. The temperature is normal. X-ray examination reveals a hemispherical protrusion in the area of the right dome of the diaphragm. What disease can be thought of in this case? Select one:
A. Liver echinococcosis
B. Metastatic tumor
C. Hepatocellular cancer
D. Liver abscess
E. Subphrenic abscess
The correct answer is: Echinococcosis of the liver
Explanation
The most likely diagnosis for the 41-year-old patient who complains of heaviness in the area of the right hypochondrium, has a palpable liver edge 3 cm below the costal arch, a hemispherical protrusion in the area of the right dome of the diaphragm on X-ray examination, and a history of causeless urticaria and itching of the skin a year ago, is liver echinococcosis. Liver echinococcosis is a parasitic infection caused by the tapeworm Echinococcus granulosus. The infection typically occurs through ingestion of contaminated food or water and can lead to the development of cysts in the liver. The symptoms of liver echinococcosis can vary depending on the size and location of the cysts and may include abdominal pain, nausea, vomiting, and hepatomegaly. However, some patients may be asymptomatic. In this case, the presence of a palpable liver edge, a hemispherical protrusion in the area of the right dome of the diaphragm on X-ray examination, and a history of causeless urticaria and itching of the skin a year ago are all consistent with liver echinococcosis. The parasitic cysts can cause pressure on adjacent structures, leading to symptoms such as abdominal discomfort and diaphragmatic irritation, and may also provoke an immune response that can result in urticaria and itching of the skin. Metastatic tumor, hepatocellular cancer, liver abscess, and subphrenic abscess are less likely in this case, as they do not typically present with the characteristic symptoms and physical examination findings of liver echinococcosis. Therefore, based on the information provided in the question, the most likely diagnosis for the 41-year-old patient who complains of heaviness in the area of the right hypochondrium, has a palpable liver edge 3 cm below the costal arch, a hemispherical protrusion in the area of the right dome of the diaphragm on X-ray examination, and a history of causeless urticaria and itching of the skin a year ago, is liver echinococcosis. Urgent medical attention is needed to confirm the diagnosis and initiate appropriate treatment, which may include medication to shrink or kill the cysts, or surgical removal of the cysts. |
50. After hypothermia, the patient was admitted to the reception department of the hospital with complaints of sharp pain in the lower back, an increase in body temperature to 38°C. Took aspirin. In the blood: leukocytes – $10.5cdot10^9$/l, eos.- 5%, pal.- 8%, segm.- 51%, lymph.- 32%, mon.- 4%, erythrocyte sedimentation rate – 28 mm/h. In the urine: protein – 0.6 g/l, leukocytes – cover the entire field of vision, a lot of mucus. What is the most likely diagnosis? Select one:
A. Acute glomerulonephritis
B. Chronic pyelonephritis
C. Tubulointerstitial nephritis
D. Acute pyelonephritis
E. Subacute malignant glomerulonephritis
The correct answer is: Acute pyelonephritis
Explanation
The most likely diagnosis for the patient who was admitted to the hospital with complaints of sharp pain in the lower back, an increase in body temperature to 38°C, leukocytosis, and pyuria, and who has a history of hypothermia, is acute pyelonephritis. Acute pyelonephritis is a bacterial infection of the renal pelvis and the kidney parenchyma. It typically presents with fever, flank pain, and dysuria, as well as leukocytosis and pyuria on laboratory tests. Risk factors for acute pyelonephritis include urinary tract obstruction, vesicoureteral reflux, and immunosuppression. In this case, the patient has a history of hypothermia, which may have predisposed them to infection, and is exhibiting symptoms and laboratory findings consistent with acute pyelonephritis, including fever, sharp pain in the lower back, leukocytosis with neutrophilic predominance, and pyuria with a lot of mucus. Acute glomerulonephritis, chronic pyelonephritis, tubulointerstitial nephritis, and subacute malignant glomerulonephritis are less likely in this case, as they do not typically present with the characteristic symptoms and laboratory findings of acute pyelonephritis. Acute glomerulonephritis is a type of kidney inflammation that can occur after a streptococcal infection and typically presents with hematuria, proteinuria, and edema. Chronic pyelonephritis is a long-standing infection of the kidney parenchyma and renal pelvis, which can lead to renal scarring and functional impairment. Tubulointerstitial nephritis is an inflammatory condition of the kidney tubules and interstitium, which can be caused by drugs, autoimmune disorders, or infections. Subacute malignant glomerulonephritis is a rare and aggressive form of glomerulonephritis, which typically presents with rapidly progressive renal failure. Therefore, based on the information provided in the question, the most likely diagnosis for the patient who was admitted to the hospital with complaints of sharp pain in the lower back, an increase in body temperature to 38°C, leukocytosis, and pyuria, and who has a history of hypothermia, is acute pyelonephritis. Urgent medical attention is needed to confirm the diagnosis and initiate appropriate treatment, which typically includes antibiotics and supportive care to manage pain and fever. |